Michael's maximizing mania

Algebra Level 4

If x , y , x, y, and z z are positive real numbers such that x + 2 y + 3 z = 12 , x+2y+3z = 12, what is the largest possible value of 3 2 x y 4 z \dfrac{3}{2}xy^4z ?

This problem is posed by Michael T .


The answer is 512.

This section requires Javascript.
You are seeing this because something didn't load right. We suggest you, (a) try refreshing the page, (b) enabling javascript if it is disabled on your browser and, finally, (c) loading the non-javascript version of this page . We're sorry about the hassle.

17 solutions

Sean Elliott
Dec 22, 2013

By the arithmetic-geometric mean inequality, 2 x + y + y + y + y + 6 z 6 ( 2 x y 4 6 z ) 1 6 \frac{2x+y+y+y+y+6z}{6}\ge(2xy^46z)^{\frac{1}{6}} Plugging in the expression x + 2 y + 3 z = 12 x+2y+3z=12 , we get 24 6 = 4 ( 2 x y 4 6 z ) 1 6 \frac{24}{6}=4\ge(2xy^46z)^{\frac{1}{6}} Raising each side to the 6 t h 6^{th} power yields 4 6 = 2 12 2 x y 4 6 z 4^6=2^{12}\ge2xy^46z or 2 10 3 x y 4 z \frac{2^{10}}{3}\ge xy^4z Thus 3 2 x y 4 z 3 2 2 10 3 = 2 9 = 512 \frac{3}{2}xy^4z\le\frac{3}{2}\cdot\frac{2^{10}}{3}=2^9=512 However, we must check that this value can be obtained. In the AM-GM inequality, equality is obtained if each term is equal. Thus we have 2 x = y = 6 z 2x=y=6z . Plugging this into the original equation gives y 2 + 2 y + y 2 = 12 y = 4 ( x , y , z ) = ( 2 , 4 , 2 3 ) \frac{y}{2}+2y+\frac{y}{2}=12\Rightarrow y=4\Rightarrow(x,y,z)=(2,4,\frac{2}{3}) Now we have 3 2 x y 4 z = 3 2 ( 2 ) ( 4 ) 4 ( 2 3 ) = 2 9 \frac{3}{2}xy^4z=\frac{3}{2}(2)(4)^4(\frac{2}{3})=2^9 So this value can be obtained, and 512 \boxed{512} is our answer.

Lagrange multipliers imply that the maximum is achieved when x = 2; y = 4 and z = 2/3. That max value is 512! :)

Мирослав Маринов - 7 years, 5 months ago

Log in to reply

I also used Lagrange multipliers to solve this problem.

Tunk-Fey Ariawan - 7 years, 4 months ago

Great use of AM-GM!

Billy Sugiarto - 7 years, 4 months ago
Kishlaya Jaiswal
Dec 22, 2013

This problem is completely observation based.

We need to apply AM-GM inequality but not in a direct manner. We need to do some mathematical jugglery with x + 2 y + 3 z x+2y+3z so as to get the maximum value of 3 2 x y 4 z \frac{3}{2}xy^4z

Observe that, we can write x + 2 y + 3 z = x + y 2 + y 2 + y 2 + y 2 + 3 z = 12 x+2y+3z = x+ \frac{y}{2}+ \frac{y}{2}+ \frac{y}{2}+ \frac{y}{2} + 3z = 12

Now, we can apply AM-GM inequality to easily get our answer

x + y 2 + y 2 + y 2 + y 2 + 3 z 6 ( 3 16 x y 4 z ) 1 6 \frac{x+ \frac{y}{2}+ \frac{y}{2}+ \frac{y}{2}+ \frac{y}{2} + 3z}{6} \geq (\frac{3}{16}xy^4z)^{\frac{1}{6}}

( 3 16 x y 4 z ) 1 6 12 6 = 2 (\frac{3}{16}xy^4z)^{\frac{1}{6}} \leq \frac{12}{6} = 2

3 16 x y 4 z 2 6 = 64 \frac{3}{16}xy^4z \leq 2^6 = 64

3 2 x y 4 z 64 × 8 = 512 \frac{3}{2}xy^4z \leq 64\times8 = 512

Hence, the maximum value of 3 2 x y 4 z \frac{3}{2}xy^4z is 512 \boxed{512}

Is important to explain whether or not 512 is attainable!

Jorge Tipe - 7 years, 5 months ago

Log in to reply

I think it would always be attainable.. Can you please give a counter example where a particular value is not attainable using the AM-GM inequality?

Mridul Sachdeva - 7 years, 5 months ago

Log in to reply

The only reason I could see is that A M G M AM-GM is applied to non-negative reals while the problem statement asks for positive reals. Otherwise I see no reason why equality could not be attainable, when x = y 2 = 3 z x = \frac{y}{2} = 3z .

This would give a solution of ( 2 , 4 , 2 3 ) (2, 4, \frac{2}{3}) .

Michael Tong - 7 years, 5 months ago

Log in to reply

@Michael Tong I use Lagrange multipliers to solve this problem.

Tunk-Fey Ariawan - 7 years, 4 months ago

I mean, it is sufficient to say "Equality occurs when x=y/2=3z=12/6". Maybe in this problem is obvious that 512 is attainable after using AM-GM, just mention when equality occurs.

Jorge Tipe - 7 years, 5 months ago

Log in to reply

@Jorge Tipe To explain why one has to mention that 512 512 can be achieved, consider the same problem, but with an additional constraint x z x\leq z . Then the same AM-GM argument as in the original problem gives an upper bound of 512. 512. However, it is no longer the maximum, because it is not achieved. The correct maximum for this modified question is actually smaller.

Alexander Borisov - 7 years, 5 months ago

Log in to reply

@Alexander Borisov Ok.. Thanks :)

Mridul Sachdeva - 7 years, 5 months ago
Nishant Sharma
Dec 22, 2013

Since we see y 4 y^4 so we will manipulate 2 y 2y as y 2 \displaystyle\frac{y}{2} + y 2 + \displaystyle+\frac{y}{2}+ y 2 + \displaystyle\frac{y}{2}+ y 2 \displaystyle\frac{y}{2} . Now employing AM-GM inequality as the numbers are positive, we have

x + y 2 + y 2 + y 2 + y 2 + 3 z 6 ( 3 x y 4 z 16 ) 1 6 \displaystyle\frac{x+\frac{y}{2}+\frac{y}{2}+\frac{y}{2}+\frac{y}{2}+3z}{6}\geq\left(\frac{3xy^4z}{16}\right)^{\frac{1}{6}}

Substituting x + 2 y + 3 z \displaystyle\,x+2y+3z with 12 12 and raising both sides to 6 t h \displaystyle\,6^{th} power we have 3 2 x y 4 z 512 \frac{3}{2}xy^4z\leq{512} .

Akshaj Kadaveru
Dec 23, 2013

We know x + 2 y + 3 z = 12 x + 2y + 3z = 12 so x + 1 2 y + 1 2 y + 1 2 y + 1 2 y + 3 z = 12 x + \dfrac12 y + \dfrac12 y + \dfrac12 y + \dfrac12 y + 3z = 12 . From AM-GM, we know

2 x + 1 2 y + 1 2 y + 1 2 y + 1 2 y + 3 z 6 x ( 1 2 y ) 4 3 z 6 3 x 4 z 16 6 2 \ge \dfrac{x + \dfrac12 y + \dfrac12 y + \dfrac12 y + \dfrac12 y + 3z}{6} \ge \sqrt[6]{x \cdot \left(\dfrac12 y\right)^4 \cdot 3z} \ge \sqrt[6]{\dfrac{3x^4z}{16}} Therefore 64 3 x 4 z 16 64 \ge \dfrac{3x^4z}{16} so 512 = 64 8 3 2 x 4 z 512 = 64 \cdot 8 \ge \dfrac32 x^4z . Equality occurs when x = 2 , y = 4 , z = 2 3 x = 2, \ y=4, \ z=\dfrac23 so our answer is 512 \boxed{512} .

Ricky Theising
Dec 22, 2013

Notice that we can "break" up the given equation into

x + y 2 + y 2 + y 2 + y 2 + 3 z = 12 x + \frac{y}{2} + \frac{y}{2} + \frac{y}{2} + \frac{y}{2} + 3z = 12

But by AM-GM, we have

x + y 2 + y 2 + y 2 + y 2 + 3 z x y 4 16 3 z 6 x + \frac{y}{2} + \frac{y}{2} + \frac{y}{2} + \frac{y}{2} + 3z \geq \sqrt[6]{x\frac{y^4}{16}3z}

12 x y 4 16 3 z 6 12 \geq \sqrt[6]{x\frac{y^4}{16}3z}

which is equivalent to

1024 3 x y 4 z \frac{1024}{3} \geq xy^{4}z

multiplying by 3 2 \frac{3}{2} yields

512 3 2 x y 4 z 512 \geq \frac{3}{2}xy^{4}z

this means that the maximum value of 3 2 x y 4 z \frac{3}{2}xy^{4}z is 512 512 .

Is important to explain whether or not 512 is attainable!

Jorge Tipe - 7 years, 5 months ago

Log in to reply

512 is attainable when x=y/2=3z=12/6. I don't think this result needs to be explained, because it is a standard result of the AM-GM inequality.

Meet Udeshi - 7 years, 5 months ago

Log in to reply

It is obvius, but important, I think. It is sufficient to say "Equality occurs when x=y/2=3z=12/6".

Jorge Tipe - 7 years, 5 months ago
Tunk-Fey Ariawan
Feb 3, 2014

It took more than an hour for me to solve this problem because I have to learn the method of Lagrange multipliers first. I've never heard a such thing as Lagrange multipliers before. Nice problem since I get a new kind of knowledge.

Exactly the same situation as mine

Eric Jan Escober - 4 years, 6 months ago

Log in to reply

I had the idea of.using AM-GM inequality but I dont seem to have a good grasp of it yet. :-/

Eric Jan Escober - 4 years, 6 months ago

Using the method of Lagrange multipliers :

Note that the values of ( x , z , y ) (x, z, y) that give the maximum of 3 2 x y 4 z \frac{3}{2}xy^4z also give the maximum of x y 4 z xy^4z and of ln ( x y 4 z ) = ln ( x ) + 4 ln ( y ) + ln ( z ) \ln(xy^4z) = \ln(x) +4 \ln(y)+ \ln(z) since x , y , z x,y,z are all positive.

We set up the Lagrangian

L = ln ( x ) + 4 ln ( y ) + ln ( z ) λ ( x + 2 y + 3 z 12 ) \mathcal{L} = \ln(x) +4 \ln(y)+ \ln(z) - \lambda(x+2y+3z-12)

Now take all partial derivative and set to zero.

L x = 1 x λ = 0 x = 1 λ \dfrac{\partial \mathcal{L}}{\partial x} = \frac{1}{x} - \lambda =0 \iff x = \frac{1}{\lambda}

L y = 4 y 2 λ = 0 2 y = 4 λ \dfrac{\partial \mathcal{L}}{\partial y} = \frac{4}{y} - 2\lambda =0 \iff 2y = \frac{4}{\lambda}

L z = 1 z 3 λ = 0 3 z = 1 λ \dfrac{\partial \mathcal{L}}{\partial z} = \frac{1}{z} - 3\lambda =0 \iff 3z = \frac{1}{\lambda}

L λ = x 2 y 3 z + 12 = 0 \dfrac{\partial \mathcal{L}}{\partial \lambda} = -x-2y-3z+12 =0

Plug into the last partial derivative to get value of λ \lambda and we find ( x , z , y ) = ( 2 , 4 , 2 / 3 ) (x, z, y) = ( 2,4,2/3) .

Evaluating 3 2 x y 4 z \frac{3}{2}xy^4z at those values gives 512 \boxed{512} .

Aryan C.
Dec 24, 2013

As x y z are positive reals we can apply AM-GM inequality. x +4 (y/2)+3z>(3/16 xy^4z)^1/6 On solving we get the required value <= 512. Hence its maximum value is 512.

Raj Magesh
Dec 24, 2013

Whenever I see questions like this, I start out by trying the AM-GM inequality. Since we want a product of x x , y 4 y^{4} and z z on the Geometric Mean side, we rewrite the given equation to include 4 terms involving y y .

x + 2 y + 3 z = x + y 2 + y 2 + y 2 + y 2 + 3 z = 12 x + 2y + 3z = x + \dfrac{y}{2} + \dfrac{y}{2} + \dfrac{y}{2} + \dfrac{y}{2} + 3z = 12

Now, taking AM-GM, we get:

x + y 2 + y 2 + y 2 + y 2 + 3 z 6 ( x y 2 y 2 y 2 y 2 3 z ) 1 6 \dfrac{x + \dfrac{y}{2} + \dfrac{y}{2} + \dfrac{y}{2} + \dfrac{y}{2} + 3z}{6} \ge \left(x*\dfrac{y}{2}*\dfrac{y}{2}*\dfrac{y}{2}*\dfrac{y}{2}*3z\right)^{\frac{1}{6}}

12 6 ( 3 x y 4 z 16 ) 1 6 \dfrac{12}{6} \ge \left(\dfrac{3xy^{4}z}{16}\right)^{\frac{1}{6}}

2 6 3 x y 4 z 16 2^{6} \ge \dfrac{3xy^{4}z}{16}

8 × 2 6 8 × ( 3 x y 4 z 16 ) 8 \times 2^{6} \ge 8 \times \left(\dfrac{3xy^{4}z}{16}\right)

512 3 2 x y 4 z 512 \ge \dfrac{3}{2}xy^{4}z

Hence, the maximum value of 3 2 x y 4 z \dfrac{3}{2}xy^{4}z is 512 \boxed{512}

Joel Tan
Dec 23, 2013

The first linear equation is equal to x + 1 2 y + 1 2 y + 1 2 y + 1 2 y + 3 z = 12. x+\frac{1}{2}y+\frac{1}{2}y+\frac{1}{2}y+\frac{1}{2}y+3z=12. By AM-GM inequality, x + 1 2 y + 1 2 y + 1 2 y + 1 2 y + 3 z 6 \frac{x+\frac{1}{2}y+\frac{1}{2}y+\frac{1}{2}y+\frac{1}{2}y+3z}{6} is greater than or equal to 6 3 16 x y 4 z ^{6}\sqrt{\frac{3}{16}xy^{4}z} (the product of the six terms on the numerator on the left hand side.) From this equation it can be seen that the largest possible value is 8 × 2 6 = 512 8 \times 2^{6}=512 which can be achieved when x = y 2 = 3 z x=\frac{y}{2}=3z ( x = 2 , y = 4 , z = 2 3 ) (x=2, y=4, z=\frac{2}{3})

Sorry for the error: "on the left hand side" should be referring to the second expression (the one with the 6 in the denominator)

Joel Tan - 7 years, 5 months ago
Suryansh Tiwari
Feb 10, 2014

A.M . > G.M

if positive real nos are given ,then atleast one times we should think about the A.M>=G.M,let x,y/2,y/2,y/2,y/2,3z , there are six positive no and after the multiplication of this nos we are getting xy^4z factor.then here we can apply A.M>=G.M (x+y/2+y/2+y/2+y/2+3z)/6>=6th root of (x y/2 y/2 y/2 y/2*3z).After solving this we can get the ANS.And the ans is 512.

Kalyan Pakala
Jan 16, 2014

Y cannot be 5 as the denominator is 2. therefore y has to be 4.taking z to be divisible by 3, z=1/3=.>x=3 and the product is 192. let z=2/3 and x=2 and the product is 512. Lagrangian approach is methodical but I took the intuitive route.

x + 2 y + z x+2y+z can be written as

x + ( 1 / 2 ) y + ( 1 / 2 ) y + ( 1 / 2 ) y + ( 1 / 2 ) y + 3 z x + (1/2)y + (1/2)y + (1/2)y + (1/2)y + 3z

By AM - GM Inequality, we have

x + ( 1 / 2 ) y + ( 1 / 2 ) y + ( 1 / 2 ) y + ( 1 / 2 ) y + 3 z 6 3 16 x y 4 z 6 x + (1/2)y + (1/2)y + (1/2)y + (1/2)y + 3z \ge 6\sqrt[6]{\frac{3}{16}xy^{4}z}

But x + 2 y + z = x + ( 1 / 2 ) y + ( 1 / 2 ) y + ( 1 / 2 ) y + ( 1 / 2 ) y + 3 z = 12 x+2y+z = x + (1/2)y + (1/2)y + (1/2)y + (1/2)y + 3z = 12 ; hence

6 3 16 x y 4 z 6 12 6\sqrt[6]{\frac{3}{16}xy^{4}z}\le12

or

3 16 x y 4 z 6 2 \sqrt[6]{\frac{3}{16}xy^{4}z}\le2

It follows that

3 2 x y 4 z 2 9 \frac{3}{2}xy^{4}z\le2^{9}

Which means that the maximum value of 3 2 x y 4 z \frac{3}{2}xy^{4}z must be 2 9 2^{9} or 512 \boxed{512}

Shivam Khosla
Dec 23, 2013

Split 2y into y/2,+y/2+y/2+y/2....now apply am>gm to 6 quantities which also include x and 3z

Siddharth Shah
Dec 23, 2013

first of all our focus goes to the "Largest possible value", hence we think of the AM>=GM concept. Now, to get the desired 3/2 x y^4 z we break 2y as y/2 + y/2 + y/2 + y/2 after which we will apply the AM>= GM for x, the broken y's and 3z therefore, we have x + y / 2 + y / 2 + y / 2 + y / 2 + 3 z 6 \frac{x + y/2 +y/2+y/2+y/2 +3z}{6} = (3/16 x y^4 z)^1/6) we know that the LHS =12. Hence on solving we get 3/2 x y 4 y^{4} z <=512 Hence the answer 512 \boxed{512}

Amit Kharwandikar
Dec 22, 2013

x+2y+3z=12= (x+y/2+y/2+y/2+y/2+3z)..... <1> using AM>OR=GM in <1>, (x+y/2+y/2+y/2+y/2+3z)/6 >or= (x* y^4/16 3z)^1/6 2^6 >or= x (y^4)/16 3z 1024/3 >or= x y^4 z therefore, 3/2(x y^4 z) <or= (3/2) (1024/3) 3/2(x* y^4 *z) <or= 512

0 pending reports

×

Problem Loading...

Note Loading...

Set Loading...